JEE Main & Advanced Mathematics Vector Algebra Question Bank Modulus of vector Algebra of vectors

  • question_answer
    A unit vector a makes an angle \[\frac{\pi }{4}\] with z-axis. If \[\mathbf{a}+\mathbf{i}+\mathbf{j}\] is a unit vector, then a is equal to                                 [IIT 1988]

    A) \[\frac{\mathbf{i}}{2}+\frac{\mathbf{j}}{2}+\frac{\mathbf{k}}{\sqrt{2}}\]             

    B) \[\frac{\mathbf{i}}{2}+\frac{\mathbf{j}}{2}-\frac{\mathbf{k}}{\sqrt{2}}\]

    C) \[-\frac{\mathbf{i}}{2}-\frac{\mathbf{j}}{2}+\frac{\mathbf{k}}{\sqrt{2}}\]              

    D) None of these

    Correct Answer: C

    Solution :

    Let \[\mathbf{a}=l\mathbf{i}+m\mathbf{j}+n\mathbf{k},\] where \[{{l}^{2}}+{{m}^{2}}+{{n}^{2}}=1.\] a makes an angle \[\frac{\pi }{4}\] with \[z-\]axis. \[\therefore \,\,n=\frac{1}{\sqrt{2}},\] \[{{l}^{2}}+{{m}^{2}}=\frac{1}{2}\]                                                  ?..(i) \[\therefore \,\,\mathbf{a}=l\,\mathbf{i}+m\,\mathbf{j}+\frac{\mathbf{k}}{\sqrt{2}}\] \[\mathbf{a}+\mathbf{i}+\mathbf{j}=(l+1)\mathbf{i}+(m+1)\mathbf{j}+\frac{\mathbf{k}}{\sqrt{2}}\] Its magnitude is 1, hence \[{{(l+1)}^{2}}+{{(m+1)}^{2}}=\frac{1}{2}\]   .....(ii) From (i) and (ii), \[2lm=\frac{1}{2}\Rightarrow l=m=-\frac{1}{2}\] Hence \[\mathbf{a}=-\frac{\mathbf{i}}{2}-\frac{\mathbf{j}}{2}+\frac{\mathbf{k}}{\sqrt{2}}\].


You need to login to perform this action.
You will be redirected in 3 sec spinner